- Thu Jan 21, 2016 12:00 am
#40930
Complete Question Explanation
(The complete setup for this game can be found here: lsat/viewtopic.php?t=1864)
The correct answer choice is (E)
This question is tailor-made to use solutions produced in other questions. As such, this would be a good question to skip, and then return to after all of the other questions have been answered. Consider some of the information available in other questions:
Proving that W could be used in the sixth year is superfluous, and thus answer choice (E) is correct.
As a side observation, note that if W (which has a restriction relative to V) can go in any space, N (which has no restrictions) must be free to go in any space, as well.
(The complete setup for this game can be found here: lsat/viewtopic.php?t=1864)
The correct answer choice is (E)
This question is tailor-made to use solutions produced in other questions. As such, this would be a good question to skip, and then return to after all of the other questions have been answered. Consider some of the information available in other questions:
- Question #17. The solution in the correct answer shows that W can be used in the third year. This information eliminates answer choice (D), which does not contain 3.
Question #19 and question #21. The solutions in these question show that W could be used in the second year. This information eliminates answer choice (A), which does not contain 2.
Question #22. The correct answer places N in the second year, which, from the split-block inferred during the setup, then places W in the fifth year. This information eliminates answer choice (C) and answer choice (D) (again), neither of which contain 5.
Proving that W could be used in the sixth year is superfluous, and thus answer choice (E) is correct.
As a side observation, note that if W (which has a restriction relative to V) can go in any space, N (which has no restrictions) must be free to go in any space, as well.
You do not have the required permissions to view the files attached to this post.